Difference between revisions of "2017 AMC 12A Problems/Problem 6"

(Added See Also)
(See Also)
Line 10: Line 10:
 
==See Also==
 
==See Also==
 
{{AMC12 box|year=2017|ab=A|num-b=5|num-a=7}}
 
{{AMC12 box|year=2017|ab=A|num-b=5|num-a=7}}
 +
{{MAA Notice}}

Revision as of 18:17, 8 February 2017

Problem

Joy has $30$ thin rods, one each of every integer length from $1 \text{ cm}$ through $30 \text{ cm}$. She places the rods with lengths $3 \text{ cm}$, $7 \text{ cm}$, and $15 \text{cm}$ on a table. She then wants to choose a fourth rod that she can put with these three to form a quadrilateral with positive area. How many of the remaining rods can she choose as the fourth rod?


$\textbf{(A)}\ 16 \qquad\textbf{(B)}\ 17 \qquad\textbf{(C)}\ 18 \qquad\textbf{(D)}\ 19  \qquad\textbf{(E)}\ 20$

Solution

See Also

2017 AMC 12A (ProblemsAnswer KeyResources)
Preceded by
Problem 5
Followed by
Problem 7
1 2 3 4 5 6 7 8 9 10 11 12 13 14 15 16 17 18 19 20 21 22 23 24 25
All AMC 12 Problems and Solutions

The problems on this page are copyrighted by the Mathematical Association of America's American Mathematics Competitions. AMC logo.png